LSAT and Law School Admissions Forum

Get expert LSAT preparation and law school admissions advice from PowerScore Test Preparation.

User avatar
 Dave Killoran
PowerScore Staff
  • PowerScore Staff
  • Posts: 5853
  • Joined: Mar 25, 2011
|
#41356
Complete Question Explanation
(The complete setup for this game can be found here: lsat/viewtopic.php?t=2035)

The correct answer choice is (B)

If both F and J work during the week, then the fourth rule comes into play, and from the Not Laws that follow from a sequence in a linear game, we know that J cannot work on Monday and F cannot work on Friday. Consequently, answer choice (B) cannot occur, and is correct.

Get the most out of your LSAT Prep Plus subscription.

Analyze and track your performance with our Testing and Analytics Package.